Unique Fixed Point: Prove Contraction Map in Compact Space Has Unique Solution

  • Thread starter Thread starter Bashyboy
  • Start date Start date
  • Tags Tags
    Fixed point Point
Click For Summary
The discussion focuses on proving that a contraction map defined on a compact metric space has a unique fixed point. The user demonstrates the continuity of the function g(x) = d(f(x), x) and explores various approaches to simplify the proof, including using the composition of continuous functions. They clarify that if g(x) > 0, it leads to a contradiction regarding the minimum value of g, ultimately showing that d(f(x), x) must equal zero, confirming f(x) = x. The uniqueness of the fixed point is established by demonstrating that if two points are fixed, they must be equal, contradicting the contraction property. The user resolves their confusion about contradictions in the proof, concluding that the contraction condition ensures uniqueness when x and y are distinct.
Bashyboy
Messages
1,419
Reaction score
5

Homework Statement



Let ##(X,d)## be some metric space, and let ##f : X \to X## be such that ##d(f(x),f(y)) \le a d(x,y)## for every ##x,y \in X## for some ##a \in (0,1)## (such a map is a called a contraction map)

If ##f## is a contraction and ##X## is compact, show that ##f## has a unique fixed point.

Homework Equations

The Attempt at a Solution



I will first show that ##g : X \to \Bbb{R}## defined by ##g(x) = d(f(x),x)## is continuous. First note that ##d(f(x),x) - d(x,y) \le d(f(x),y)## and ##d(x,y) - d(f(y),y) \le d(f(y),x)##, as well as ##d(f(x),y) \le d(f(x),x) + d(x,y)## and ##d(f(y),x) \le d(f(y),y) + d(x,y)##. Therefore

\begin{align*}
|d(f(x),x) - d(f(y),y)| & = \left|\bigg(d(f(x),x) - d(x,y)\bigg) + \bigg(d(x,y) - d(f(y),y) \bigg) \right| \\
& \le |d(f(x),x) - d(x,y)| + |d(x,y) - d(f(y),y)| \\
& \le d(f(x),y) + d(f(y),x) \\
& \le d(f(x),x) + d(x,y) + d(f(y),y) + d(x,y) \\
& \le 2(\alpha +1)d(x,y), \\
\end{align*}

from which we can conclude ##g## is continuous. This seems right, albeit a bit roundabout. Is there a more clever way of adding in ##0##? I think there is a way of viewing it as a composition of continuous functions. Let ##e : X \to X \times X## defined by ##e(x) = (f(x),id(x)) = (f(x),x)##, which is continuous by the maps-to-products theorem. Then ##g = d \circ e##, a composition of continuous functions (this obviously answers the original question in my OP). I am still interested in a less roundabout way of using the triangle inequality.

My goal is to try and use the solution found https://math.stackexchange.com/questions/118536/prove-the-map-has-a-fixed-point I tried redefining ##g## to be ##g(x) = a d(f(x),x)## and ##g(x) = \frac{1}{a} d(f(x),x)##, but I couldn't get it to work. I think part of the problem lies in the fact that I am working with ##\le## and not strict inequality. Does anyone see a way of modifying the solution found in the link above.
 
Physics news on Phys.org
If ≤ is your problem, can you consider b, where a<b<1 and get the strict < that will work for you?
EDIT: Get the strict < as long as d(x,y) >0.
 
Last edited:
Ah! Blast! If ##a \in (0,1)##, then ##a d(x,y) < d(x,y)## so that ##d(f(x),f(y)) \le a d(x,y) < d(x,y)##, and so we can easily use the method in the given link. Therefore, since ##g## is continuous and ##X## is compact, it achieves a minimum over ##X## at, say, the point ##x \in X##; hence, ##g(x) \le g(y)## for every ##y \in X##. Now, if ##0 = g(x) = d(f(x),x)##, then we are finished. However, if ##g(x) > 0##, then ##d(f(f(x),f(x)) \le ad(f(x),x) < d(f(x),x)## which contradicts the fact that the minimum is attained at ##x##. Therefore ##d(f(x),x)=0## and so ##f(x) = x##.

The only thing that confuses me is, isn't ##d(f(f(x),f(x)) \le ad(f(x),x) < d(f(x),x)## a contradiction especially when ##d(f(x),x)=0## or ##f(x)=x##, since this would say ##d(f(f(x)),f(x)) < 0##? Also. I just realized that I didn't prove uniqueness; I'll have to think about that one some more.
 
D'oh! Uniqueness is trivial. Suppose that ##x## and ##y## are both fixed points. Then ##f(x) = x## and ##f(y) = y##. Therefore ##d(f(x),f(y)) = d(x,y)## by mere substitution; but this contradicts the fact that ##d(f(x),f(y)) < d(x,y)## for all ##x, y \in X##.

I still have that worry I expressed in my last post. Namely, if ##d(f(x),x) > 0##, then ##d(f(f(x)),f(x) < d(f(x),x)## contradicts minimality. This forces ##d(f(x),x) = 0##. However, we still get a contradiction even in this case; the contradiction that ##d(f(f(x)),f(x) < d(f(x),x) = 0## which says that ##d(f(f(x)),f(x)## is negative. Where am I going wrong? I basically using Davide Giraudo's proof, found in the link I gave above; his answer is five years old and no else has seen this issue, so I must be making a mistake somewhere...
 
Bump!
 
I figured it out: ##d(f(x),f(y)) < d(x,y)## if and only if $x \neq y##.
 
  • Like
Likes FactChecker
Question: A clock's minute hand has length 4 and its hour hand has length 3. What is the distance between the tips at the moment when it is increasing most rapidly?(Putnam Exam Question) Answer: Making assumption that both the hands moves at constant angular velocities, the answer is ## \sqrt{7} .## But don't you think this assumption is somewhat doubtful and wrong?

Similar threads

Replies
2
Views
2K
  • · Replies 18 ·
Replies
18
Views
3K
  • · Replies 5 ·
Replies
5
Views
1K
  • · Replies 1 ·
Replies
1
Views
2K
  • · Replies 12 ·
Replies
12
Views
2K
  • · Replies 0 ·
Replies
0
Views
362
  • · Replies 5 ·
Replies
5
Views
2K
  • · Replies 2 ·
Replies
2
Views
2K
Replies
9
Views
3K
  • · Replies 39 ·
2
Replies
39
Views
6K